Cases Sobra

28
1 Burgos v. Chief of Staff Placer v. Villanueva In relation to the assassination of the representative of Masbate and his security escorts, the MTC of Masbate, after the presentation of affidavits and answers of the prosecution’s witnesses, concluded that probable cause existed for issuance of a warrant of arrest against Vicente Lim and company. When the hearing of the case was transferred to Makati RTC and upon petition of spouses Lim for transmission of initial records of the preliminary investigation, the respondent Judge concluded that probable cause existed due to the declaration made by two competent officers – the MTC of Masbate and the Fiscal. Facts: 1. The Congressman of the municipality of Masbate, Masbate (Moises Espinos, Sr.) and his security escorts (Provincial Guards Antonio Cortes, Gaspar Amaro and Artemio Fuentes) were attacked and killed by a lone assassin. One security escort (Dante Siblante) survived the assassination plot but suffered a gunshot wound. 2. After an investigation of the incident, the designated investigator (Harry Tantiado of the PC Criminal Investigation Service at Camp Bagong Ibalon, Legazpi City) filed an amended complaint accusing Vicente Lim, Sr., Mayor Susana Lim of Masbate, Jolly T. Fernandez, Florencio T. Fernandez, Jr., Nonilon A. Bagalihog, Mayor Nestor C. Lim and Mayor Antonio Kho of the crime of multiple murder and frustrated murder. 3. The Municipal Trial Court of Masbate, upon weighing the affidavits and answers given by the witnesses for the prosecution during the preliminary investigation in searching questions and answers, concluded that a probable cause had been established for the issuance of a warrant of arrest against the Lim, et.al. The recommended amount for bail of each of the accused was Php 200,000.00. Except for Cabarles, all of the accused posted bail. 4. The Fiscal (Antonio Alfane), a month after the entire records of the case (261 pages) were transmitted, issued a resolution which affirmed the finding of a prima facie case against Lim, et.al. but differed in the designation of the crime. He ruled that all of the accused should not only be charged with Multiple Murder with Frustrated Murder, but for a case of murder for each of the killing of the four victims and a physical injuries case for inflicting gunshot wound on the buttocks of Siblante. Said Fiscal filed with the RTC of Masbate four separate informations of murder against the 12 accused with a recommendation of no bail. 5. The hearing of the case, due to the verified petition filed by Lim with the SC, was transferred to the RTC of Makati, Branch 56 (under Judge Nemesio Felix). The Lims filed with the said court motions and manifestations, which include, among others, issue an order for transmission of the initial records of the preliminary investigation conducted in Masbate. These were denied by the respondent court for lack of merit. 6. Felix said that there exists probable cause that the “offense of multiple murder was committed… affirmed upon review by the Provincial Prosecutor... Considering that both the two competent officers to whom such duty was entrusted by law have declared the existence of probable cause, each information is complete in form and substance, and there is no visible defect on its face…” Issue: Whether or not a judge may issue a warrant of arrest without bail by simply relying on the prosecution’s certification and recommendation that a probable cause exists Ruling: 1. The issuance of a warrant is not a mere ministerial function; it calls for the exercise of judicial discretion on the part of the issuing magistrate (from Section 6, Rule 112 of the Rules of Court). Under this section, the judge must satisfy himself of the existence of probable cause before issuing a warrant or order of arrest. If on the face of the information the judge finds no probable cause, he may disregard the fiscal’s certification and require the submission of the affidavits of witnesses to aid him in arriving at a conclusion as to the existence of a probable cause. Case Digests: Searches and Seizures • Mark Justin Mooc

description

Cases

Transcript of Cases Sobra

1 Burgos v. Chief of Staff

Placer v. VillanuevaIn relation to the assassination of the representative of Masbate and his security escorts, the MTC of Masbate, after the presentation of affidavits and answers of the prosecution’s witnesses, concluded that probable cause existed for issuance of a warrant of arrest against Vicente Lim and company. When the hearing of the case was transferred to Makati RTC and upon petition of spouses Lim for transmission of initial records of the preliminary investigation, the respondent Judge concluded that probable cause existed due to the declaration made by two competent officers – the MTC of Masbate and the Fiscal.

Facts:

1. The Congressman of the municipality of Masbate, Masbate (Moises Espinos, Sr.) and his security escorts (Provincial Guards Antonio Cortes, Gaspar Amaro and Artemio Fuentes) were attacked and killed by a lone assassin. One security escort (Dante Siblante) survived the assassination plot but suffered a gunshot wound.

2. After an investigation of the incident, the designated investigator (Harry Tantiado of the PC Criminal Investigation Service at Camp Bagong Ibalon, Legazpi City) filed an amended complaint accusing Vicente Lim, Sr., Mayor Susana Lim of Masbate, Jolly T. Fernandez, Florencio T. Fernandez, Jr., Nonilon A. Bagalihog, Mayor Nestor C. Lim and Mayor Antonio Kho of the crime of multiple murder and frustrated murder.

3. The Municipal Trial Court of Masbate, upon weighing the affidavits and answers given by the witnesses for the prosecution during the preliminary investigation in searching questions and answers, concluded that a probable cause had been established for the issuance of a warrant of arrest against the Lim, et.al. The recommended amount for bail of each of the accused was Php 200,000.00. Except for Cabarles, all of the accused posted bail.

4. The Fiscal (Antonio Alfane), a month after the entire records of the case (261 pages) were transmitted, issued a resolution which affirmed the finding of a prima facie case against Lim, et.al. but differed in the designation of the crime. He ruled that all of the accused should not only be charged with Multiple Murder with Frustrated Murder, but for a case of murder for each of the killing of the four victims and a physical injuries case for inflicting gunshot wound on the buttocks of Siblante. Said Fiscal filed with the RTC of Masbate four separate informations of murder against the 12 accused with a recommendation of no bail.

5. The hearing of the case, due to the verified petition filed by Lim with the SC, was transferred to the RTC of Makati, Branch 56 (under Judge Nemesio Felix). The Lims filed with the said court motions and manifestations, which include, among others, issue an order for transmission of the initial records of the preliminary investigation conducted in Masbate. These were denied by the respondent court for lack of merit.

6. Felix said that there exists probable cause that the “offense of multiple murder was committed… affirmed upon review by the Provincial Prosecutor... Considering that both the two competent officers to whom such duty was entrusted by law have

declared the existence of probable cause, each information is complete in form and substance, and there is no visible defect on its face…”

Issue: Whether or not a judge may issue a warrant of arrest without bail by simply relying on the prosecution’s certification and recommendation that a probable cause exists

Ruling:

1. The issuance of a warrant is not a mere ministerial function; it calls for the exercise of judicial discretion on the part of the issuing magistrate (from Section 6, Rule 112 of the Rules of Court). Under this section, the judge must satisfy himself of the existence of probable cause before issuing a warrant or order of arrest. If on the face of the information the judge finds no probable cause, he may disregard the fiscal’s certification and require the submission of the affidavits of witnesses to aid him in arriving at a conclusion as to the existence of a probable cause.

2. The 1988 Amendments to the 1985 Rules on Criminal Procedure (effective on October 1, 1988) did not restore the authority of conducting preliminary investigations to Judges of RTC; said amendments did not in fact deal at all with the officers or courts having authority to conduct preliminary investigations. This does not mean, however, that RTC judges also lost the power to make a preliminary examination for the purpose of determining whether probable cause exists to justify the issuance of a warrant of arrest or search warrant. Such power, is as much a duty as it is a power, has been and remains vested in every judge by the provision of the Bill of Rights securing the people against unreasonable searches and seizures, thereby placing it beyond the competence of mere Court Rule or Statute to revoke.

3. The distinction must be made clear: while an RTC judge may no longer conduct preliminary investigations to ascertain whether there is sufficient ground for the filing of a criminal complaint or information, he retains the authority, when such a pleading is filed with his court, to determine whether there is probable cause justifying the issuance of a warrant of arrest. It might be added that this distinction accords, rather than conflicts, with the rationale of salta, because both law and rule, in restricting judges the authority to order arrest, recognize the function to be judicial in nature.

4. Preliminary investigation should be distinguished as to whether it is an investigation for the determination of a sufficient ground for the filing of the information or it is an investigation for the determination of a probable cause for the issuance of a warrant of arrest. The first kind of preliminary investigation is executive in nature, and part of the prosecution’s job. The second kind of preliminary investigation, which is more properly called preliminary examination, is judicial in nature and is lodged with the judge.

Soliven v. MakasiarThe President of the Philippines filed a complaint for libel against the petitioners, who were the publisher and columnist of the Philippine Star, based on the following statement in

Case Digests: Searches and Seizures • Mark Justin Mooc

2 Burgos v. Chief of Staff

Beltran's column of Oct. 12, 1987 totle "The Nervous Officials of the Aquino Administration": "If you recall, during the August 29 coup attempt, the President hid under her bed while the firing was going on - perhaps the first Commander-in-Chief to do so."

Facts:

1. In this case, upon the issue raised by petitioner Beltran, the constitutional provision on the issuance of warrants of arrest was called for an interpretation. Beltran wrote in the Philippine Star that “during the August 29 coup attempt, the President hid under her bed while the firing was going on.” Due to this, the President filed a libel complaint against petitioners.

2. Beltran argues that the addition of the word “personally” after the word “determined” and the deletion of the grant of authority by the 1973 Constitution to issue warrants to “other responsible officers as may be authorized by law.” This interpretation convinced him that the Constitution now requires the judge to personally examine the complainant and his witnesses in his determination of probable cause for the issuance of warrants of arrest.

Issue: Whether or not Beltran’s constitution rights were violated when the respondent RTC judge issued a warrant of arrest without personally examining the complainant and the witnesses to determine probable cause

Ruling:

1. The judge is not required to personally examine the complainant and his witnesses. What the Constitution underscores is the exclusive and personal responsibility of the issuing judge to satisfy himself of the existence of probable cause. Instead, he shall (a) personally evaluate the report and the supporting documents submitted by the fiscal regarding the existence of probable cause and, on the basis thereof, issue a warrant of arrest; or (b) if on the basis thereof he finds no probable cause, he may disregard the fiscal’s report and require the submission of supporting affidavits of witnesses to aid him in arriving at a conclusion as to the existence of probable cause. This procedure should be followed, otherwise judges would be unduly laden with the preliminary examinations and investigation of criminal complaints instead of concentrating on hearing and deciding cases filed before their courts.

2. In making the required personal determination, a Judge is not precluded from relying on the evidence earlier gathered by responsible officers. The extent of the reliance depends on the circumstances of each case and is subject to the Judge’s sound discretion.

Roan v. Gonzales, 145 SCRA 687Roan’s house was searched by virtue of a search warrant and the said search was performed by military authorities. During their search, the authorities found a Colt Magnum revolver and 18 live bullets which they confiscated and served as bases for the charge of illegal possession of firearms. However, the application of said search warrant was based on the accounts of two witnesses. The applicant did not have personal knowledge of said firearm.

Facts:

1. A search warrant was issued by respondent judge (Gonzales) on May 10, 1984. Application for the said search warrant was personally filed by PC Capt. Mauro Quillosa. Together with Quillosa were two witnesses (Esmael Morada and Jesus Tohilida), who presented to respondent judge their respective affidavits. The application was not yet subscribed and sworn to, as such respondent Judge proceeded to examine Quillosa on the contents of the application to ascertain if he knew and understood the same. Afterwards, Quillosa subscribed and swore the said application before respondent.

2. Petitioner’s (Josefino Roan) house was searched two days after the issuance of the search warrant. The said search was performed by military authorities. Despite none of the articles listed in the warrant was discovered, the officers who conducted the search found one Colt Magnum revolver and 18 live bullets which they confiscated. The said items served as bases for the charge of illegal possession of firearms against the petitioner.

Issue: Whether or not a search warrant be annulled on the ground that it violates the privacy of one person’s house

Ruling/Decision:

1. To be valid, a search warrant must be supported by probable cause to be determined by the judge or some authorized officer after examining the complainant and the witnesses he may produce. There must be a specific description of the place to be searched and the things to be seized, to prevent arbitrary and indiscriminate use of the warrant. Probable cause, as described by Judge Escolin in Burgos v. Chief of Staff, refers to “such facts and circumstances which would lead a reasonably discreet and prudent man to believe that an offense has been committed and that the objects sought in connection with the offense are in the place sought to be searched.” The probable cause must refer to only one specific offense.

2. The applicant (Capt. Quillosa) was asking for the issuance of the search warrant on the basis of mere hearsay and not of information personally known to him as required by settled jurisprudence.

3. It is axiomatic that the magistrate must be probing and exhaustive, not merely routinary or pro-forma, if the claimed probable cause is to be established. The examining magistrate must not simply rehash the contents of the affidavit but must make his own inquiry on the intent and justification of the application.

4. Prohibited articles may be seized but only as long as the search is valid. In this case, it was not because: (a) there was no valid search warrant; and (b) absent such a warrant, the right thereto was not validly waived by the petitioner. In short, the military officers who entered the petitioner’s premises had no right to be there and therefore had no right to seize the pistol and bullets.

Case Digests: Searches and Seizures • Mark Justin Mooc

3 Burgos v. Chief of Staff

People v. Bolasa y NakoboanThree police officers (Salonga, Carizon and Arenas) peeped through a small window and saw a man and a woman repacking suspected marijuana, as they were informed by an anonymous caller. The police officers entered the house and introduced themselves as police officers and thereupon confiscated the tea bags and some paraphernalia. After the examination of the tea bags, it was confirmed that same contained marijuana.

Facts:

1. PO3 Dante Salonga and PO3 Albert Carizon were informed by an anonymous caller that a man and woman were repacking prohibited drugs at a certain house in Sta. Brigida St., Karuhatan, Valenzuela. Together with SPO1 Fernando Arenas, they proceeded immediately to the house of the suspects. As they walked toward their quarry’s (prey) lair, the three were accompanied by their unnamed informer.

2. When they reached the house, they “peeped through a small window and saw one man and a woman repacking suspected marijuana.” They entered the house and introduced themselves as police officers to the occupants and thereupon confiscated the tea bags and some paraphernalia. Examination of the tea bags by NBI Forensic Chemist confirmed the suspicion that the tea bags contained marijuana. As such, Zenaida Bolasa and Roberto delos Reyes were charged with violation of Sec. 8, Art. II of RA 6425 (Dangerous Drugs Act of 1972).

3. Both denied on the witness stand ownership over the confiscated tea bags and drug implements.

4. delos Reyes claimed that he and his wife were merely tenants in Bolasa’s house and at the time he was arrested he had just arrived from work. He added that when he learned that Bolasa was repacking marijuana inside their room, he immediately ordered her to leave. As for Bolasa, she claimed that she was about to leave the house when she met a certain “Rico” and conversed with him for some time.

5. The trial court, upon finding the version of the prosecution to be plausible, convicted both accused Bolasa and delos Reyes.

6. On appeal, Bolasa asserted that the search in her residence was illegal as her arrest preceding it was illegal. She argued that the marijuana seized from her could not be properly used as evidence against her. Together with delos Reyes, Bolasa said that PO3 Carizon was not among the arresting officers, as such Carizon had no personal knowledge regarding the conduct of the arrest and the search thus making his testimony hearsay.

Ruling:

1. An arrest is lawful even in the absence of a warrant: (a) when the person to be arrested has committed, is actually committing, or is about to commit an offense in his presence; (b) when an offense has in fact been committed and he has reasonable ground to believe that the person to be arrested has committed it; and, (c) when the person to be arrested is a prisoner who has escaped from a penal

establishment or place where he is serving final judgment or temporarily confined while his case is pending, or has escaped while being transferred from one confinement to another. The manner by which accused were apprehended does not fall under any of the above-enumerated categories. From the above, the arrest is illegal.

2. It cannot be said that the objects were seized in plain view. First, there was no valid intrusion. As already discussed, accused were illegally arrested. Second, the evidence later on found to contain marijuana was not inadvertently discovered. The police officers intentionally peeped first through the window before they saw and ascertained the activities of accused-appellants inside the room. In like manner, the search cannot be recognized as a search of a moving vehicle, a consented warrantless search, a customs search or a stop and frisk; it cannot even fall under exigent and emergency circumstances, for evidence at hand is deprived of any such showing.

3. It indicates that the apprehending officers should have conducted first a surveillance considering that the entities and address of the suspected culprits were already ascertained. After conducting the surveillance and determining the existence of probable cause for arresting accused, they (the police) should have secured a search warrant prior to effecting a valid arrest and seizure. The arrest being illegal ab initio, the accompanying search was likewise illegal. Every evidence obtained during the illegal search cannot be used against accused; hence, they were acquitted.

People v. AlundayAlunday was found to have planted, cultivated and cultured marijuana fruiting tops and have in his possession an M16 Rifle without any written authority or permit. He was rendered a decision of conviction for violation of Dangerous Drugs Act, but was acquitted for reasonable doubt for violating PD 1866. Alunday contended however that he was arrested without warrant and his warrantless arrest does not fall under the circumstances contemplated by Section 5, Rule 113 of the 1985 Rules of Court.

Facts:

1. Accused (Ricardo Alunday alias “Kayad”), “without being authorized by law, and with intent to plant and cultivate,… unlawfully and feloniously” planted, cultivated and cultured marijuana fruiting tops weighing more than 750 grams. Said marijuana fruiting tops had an estimated value of Php 10 million. For this, he was charged with violation of Section 9 of RA 6425 (Dangerous Drugs Act of 1972).

2. Alunday was likewise additionally charged with violation of PD 1866 for he was found to have possessed an M16 Rifle without any written authority or permit previously acquired from authorities to carry or transport the said firearm.

3. The RTC found Alunday guilty of violating RA 6425 while he was acquitted for violating PD 1866 for reasonable doubt. This was affirmed by the Court of Appeals.

4. Accused, in his appeal, assailed his conviction for being improper and illegal, asserting that the court a quo never acquired jurisdiction over his person because

Case Digests: Searches and Seizures • Mark Justin Mooc

4 Burgos v. Chief of Staff

he was arrested without a warrant and that his warrantless arrest was not done under any of the circumstances enumerated in Section 5, Rule 113 of the 1985 Rules of Court. He insisted that the arresting officers had 3 months within which to secure a warrant from the time they received the information about an existing marijuana plantation in Mount Churyon, Sadanga in May 2000 until they effected arrest on August 2000. Also, he contended that the arresting officers’ failure to secure a warrant can never be justified by the urgency of the situation.

Ruling:

1. Section 5, Rule 113 of the Rules of Court provides that a peace officer or a private person may, without warrant, arrest a person: (a) when the person to be arrested has committed, is actually committing, or is about to commit an offense in his presence; (b) when an offense has in fact been committed and he has reasonable ground to believe that the person to be arrested has committed it; and, (c) when the person to be arrested is a prisoner who has escaped from a penal establishment or place where he is serving final judgment or temporarily confined while his case is pending, or has escaped while being transferred from one confinement to another.

2. Section 5(a) refers to arrest in flagrante delicto. In flagrante delicto means caught in the act of committing a crime. This rule, which warrants the arrest of a person without warrant, requires that the person arrested has just committed a crime, or is committing it, or is about to commit an offense, in the presence or within view of the arresting officer.

3. In the case at bar, the information was received by the Intelligence Section of the Provincial Office of the Mountain Province in May 2000 while the accused was arrested during the police raid at the plantation at Mount Churyon, Sadanga on August. This is so because the arrest was effected only after a series of validations conducted by the team to verify or confirm the report that indeed a marijuana plantation existed at the area, which was confirmed on August 2. During the day of the arrest (August 3), the arresting team of SPO1 Saipen proceeded to the marijuana plantation and Saipen saw Alunday personally cutting and gathering marijuana plants. Therefore, his arrest was legal because he was caught in flagrante delicto.

4. It is much too late in the day to complain about the warrantless arrest after a valid information has been filed, the accused arraigned, trial commenced and completed, and a judgment of conviction rendered against him. He raised the additional issue of irregularity of his arrest only during his appeal to the SC. He is, therefore, deemed to have waived such alleged defect by submitting himself to the jurisdiction of the court by his counsel-assisted plea during his arraignment; by his actively participating in the trial and by not raising the objection before his arraignment.

People v. CruzMembers of the CRIG nabbed two persons who were to sell a stolen car. After the carnap suspects were brought to the police headquarters, they led the CRIG team to the place where other members of the carnap gang were waiting. The accused, Cruz, was found to have possessed a calibre .38 paltik revolver, one live ammunition and a hand grenade in his clutch bag. He was charged with the crime of Illegal Possession of Firearms and Ammunition. He contended however that the PC officers had no warrant of arrest and that the said firearm and explosive were found when they (carnap gang) were being arrested for “carnapping” and not for illegal possession of firearm and ammunition.

Facts:

1. Eight members of CRIG, led by Lt. Noel Manabat, stationed at Camp Bagong Diwa acted on an intelligence information that on noon of May 9, 1986 a stolen car was to be sold in Magallanes, Makati. The team nabbed Romeo Fernandez and Joey Flores and brought them to headquarters where they were questioned.

2. The two carnap suspects led the CRIG team to 61 Mabituan St., Masambong, QC where they alleged the other members of the carnap gang were waiting for their shares of the proceeds from the sale of a vehicle.

3. A calibre .38 paltik revolver, one live ammunition and a hand grenade, contained in a clutch bag, were found near accused (Reynaldo Cruz alias Rene Hapon). For this reason, he was charged with the crime of Illegal Possession of Firearm and Ammunition. He denied ownership or possession of the firearm and hand grenade, as well as the bag which contained the said items. He claimed that the bag and its contents belonged to Joey Flores and was “planted” by PC operatives.

4. Cruz contended that the firearm and explosive in question cannot be used as evidence against him since the PC officers had no warrant of arrest when they entered the apartment, in violation of his constitutional rights. Moreover, he contended that the unlicensed firearm and explosive were found when they arrested the accused and his companions for “carnapping” and not for illegal possession of firearm ammunition.

Ruling:

1. The police officers failed to comply with the strictures laid down by the Court for police officers to follow in a custodial investigation especially in the waiver of constitutional rights made without the assistance or even in the presence of counsel.

Harvey v. SantiagoHarvey, together with Sherman and Del Elshout, were alien pedophiles and were caught to have possessed articles/instruments indicating that they were engaged in child prostitution. Prior to the apprehension, members of the Commission on Immigration and Deportation performed close surveillance in Pagsanjan, Laguna. Petitioners questioned the validity of their detention due to the violation of the right against unreasonable searches and seizures.

Case Digests: Searches and Seizures • Mark Justin Mooc

5 Burgos v. Chief of Staff

Facts:

1. Petitioners (Andrew Harvey, 52, John Sherman, 72, Adriaan Van Del Elshout, 58) were among the 22 alien pedophiles who were apprehended after three of close surveillance by the Commission on Immigration and Deportation agents in Pagsanjan, Laguna. They were the only ones who have chosen to face deportation.

2. Seized during petitioners’ apprehension were rolls of photo negatives and photos of suspected child prostitutes shown in salacious (lustful) poses as well as boys and girls engaged in the sex act. There were also posters and other literature advertising the child prostitutes.

3. Based from the operation report on Harvey and Sherman dated February 29, 1988, Harvey was found together with two young boys while Sherman was found with two naked boys inside his room. Meanwhile, Del Elshout, the “after mission report” dated February 27, 1988, revealed that there were two children ages 14 and 16 which subject readily accepted having been in his care and live-in for quite sometime.

4. Deportation proceedings were instituted against the petitioners for being undesirable aliens under Section 69 of the Revised Administrative Code, being pedophiles who are inimical to public morals, public health and public safety.

5. On April 4, 1988, petitioners availed of a petition for a writ of habeas corpus. They question the validity of their detention on the ground that, among others, respondent (Miriam Santiago) violated Section 2, Article III prohibiting unreasonable searches and seizures since the CID agents were not clothed with warrants of arrest, search and seizure as required by said provision.

Ruling:

1. The right against unreasonable searches and seizures as guaranteed by Article III, Section 2 of the 1987 Constitution is available to all persons, including aliens, whether accused of crime or not. One of the constitutional requirements of a valid search warrant or warrant of arrest is that it must be based upon probable cause.

2. An arrest may be effected by a peace officer or even a private person, even without warrant, when the offense has, in fact, been committed and he has personal knowledge of facts indicating that the person to be arrested has committed it. In this case, the arrest of petitioners was based on probable cause determined after close surveillance for three months during which period their activities were monitored. The existence of probable cause justified the arrest and the seizure of the photo negatives, photographs and posters without warrant.

3. That petitioners were not caught in the act does not make their arrest illegal. They were found with boys in their respective rooms, the one with Sherman being naked. Under those circumstances, the CID agents had reasonable grounds to believe that petitioners had committed pedophilia.

Bache Co. [Phil], Inc. v. RuizMisael Vera, Commissioner of Internal Revenue, wrote a letter seeking issuance for a search warrant against Bache Co. [Phil.] for violation of Section 46(a) of the NIRC and authorizing his Revenue Examiner, de Leon, to make and file the application of search warrant. The respondent judge, since he was hearing a certain case that moment when de Leon arrived the following day, requested his Deputy Clerk of Court to take the depositions of de Leon and his witness and, after his hearing of the case and the reading of the stenographer’s notes of the depositions taken, asked the de Leon’s witness to take the oath. Three days later, the agents of BIR served the warrant and seized 6 boxes of documents.

Facts:

1. On February 24, 1970, the Commissioner of Internal Revenue (Misael Vera) wrote a letter addressed to respondent (Judge Vivencio Ruiz), requesting the issuance of a search warrant against petitioners for violation of Section 46(a) of the National Internal Revenue Code and authorizing his Revenue Examiner (Rodolfo de Leon) to make and file the application for search warrant which was attached to the letter.

2. The following day, de Leon and his witness (Arturo Logronio) went to the CFI of Rizal, bringing with them: (a) respondent Vera’s letter-request, (b) application for search warrant already filled up but still unsigned by de Leon, (c) an affidavit of respondent Logronio subscribed before de Leon, (d) a deposition in printed form of Logronio already accomplished and signed by him but not yet subscribed, and (e) a search warrant already accomplished but still unsigned by respondent Judge.

3. Since the Judge was hearing a certain case, he (Ruiz) instructed his Deputy Clerk of Court to take the depositions of de Leon and Logronio. After the session’s adjournment, he (Ruiz) asked Logronio to take the oath. Prior to Logronio’s swearing in, the stenographer, upon Ruiz’ request, read to him her stenographic notes.

4. He signed de Leon’s application for search warran and Longonio’s deposition. After which, the search warrant was signed and accordingly issued.

5. Three days later, the BIR agents served the search warrant at petitioners’ offices. Petitioners’ lawyers protested the search warrant on the ground that no formal complaint or transcript of testimony was attached to the warrant. Despite this, the agents proceeded with their search which yielded six boxes of documents.

Ruling:

1. Respondent Judge failed to personally examine the complainant and his witness. The examination of the complainant and the witnesses he may produce, required by the said constitutional provision and by Sections 3 and 4, Rule 126 of the Revised Rules of Court, should be conducted by the judge himself and not by others. In the case at bar, no personal examination was conducted by respondent Judge of the complainant and his witness. While it is true that the complainant’s application for search warrant and the witness’ printed-form description were subscribed and sworn to before Ruiz, the latter (Ruiz) did not ask any question

Case Digests: Searches and Seizures • Mark Justin Mooc

6 Burgos v. Chief of Staff

whose answers could possibly be the basis for determining whether there exists probable cause. It was precisely on account of the intention of the delegates to the Constitutional Convention to make it a duty of the issuing judge to personally examine the complainant and his witnesses. More so, the reading of the stenographic notes to respondent judge did not constitute sufficient compliance with the constitutional mandate the rule; for by that manner, respondent judge did not have the opportunity to observe the demeanor of the complainant and his witness, and to propound initial and follow-up questions which the judicial mind, on account of its training, was in the best position to conceive.

2. The search warrant cannot be issued for more than one specific offense. This is in compliance to Section 3, Rule 126 of the Rules of Court which provides that no search warrant shall issue for more than one specific offense.

3. The search warrant does not particularly describe the things to be seized. A search warrant may be said to particularly describe the things to be seized when the description therein is as specific as the circumstances will ordinarily allow; or when the description expresses a conclusion of fact by which the warrant officer may be guided in making the search and seizure; or when the things described are limited to those which bear direct relation to the offense for which the warrant is being issued.

Prudente v DayritOn a Saturday, the respondent Judge Dayrit issued a search warrant as applied for by Dimagmaliw. Dimagmaliw believed that petitioner, Prudente, had in his possession firearms and ammunitions found in the ground and second floors of Polytechnic University of the Philippines. The search warrant was enforced the following day. Found in the drawer of a cabinet inside the washroom of Dr. Prudente’s office was a bulging brown envelope with 3 live fragmentation hand grenades, each wrapped with old newspapers.

Facts:

1. Petitioner (Nemesio Prudente) was implicated for having violated PD 1866 (Illegal Possession of Firearms). As alleged by P/Major Alladin Dimagmaliw when he applied for a search warrant in the sala of Judge Abelardo Dayrit of the RTC Manila, Prudente may be found at the Polytechnic University of the Philippines where he was keeping and concealing firearms, explosive, handgrenades and ammunition, specifically at the (a) Offices of the Department for Military Science and Tactics at the ground floor and other rooms at the ground floor and (b) Office of the President , Dr. Nemesio Prudente at PUP, 2nd floor and other rooms at the 2nd floor.

2. Dimagmaliw believes that a search warrant should be issued to enable him or any agent of the law to take possession and bring to the court the following properties: (a) M16 armalites with ammunitions, (b) .38 and .45 caliber handguns and pistols, (c) explosives and handgrenades, and (d) assorted weapons with ammunition.

3. On the same day (October 31, 1987), the respondent Judge (Dayrit) issued search warrant. The following day (Sunday), with some 200 West Police Department operatives, the search warrant was enforced.

4. Meanwhile, a member of the searching team (Ricardo Abando) alleged in his affidavit that he found in the drawer of a cabinet inside the washroom of Dr. Prudente’s office a bulging brown envelope with 3 live fragmentation hand grenades separately wrapped with old newspapers.

5. Petitioner however moved to quash the search warrant on grounds that (a) the complainant’s lone witness (Lt. Florenio Angeles) had no personal knowledge of the facts which formed the basis for the issuance of the search warrant, (b) examination of the said witness was not in the form of searching questions and answers, (c) the search warrant was a general warrant for the reason that it did not particularly describe the place to be searched and that it failed to charge one specific offense, and (d) the search warrant was issued in violation of Circular No. 19 of the SC in that the complainant failed to allege under oath that the issuance of the search warrant on a Saturday was urgent.

Ruling:

1. For a valid search warrant to issue, there must be probable cause which is to be determined personally by the judge after examination under oath or affirmation of the complainant and the witnesses he may produce, and particularly describing the place to be searched and the persons or things to be seized. The probable cause must be in connection with one specific offense and the judge must, before issuing the warrant, personally examine in the form of searching questions and answers, in writing and under oath, the complainant and any witness he may produce, on facts personally known to them and attach to the record their sworn statements together with any affidavits submitted.

2. As held in Alvarez v. CFI, the true test of sufficiency of a deposition or affidavit to warrant issuance of a search warrant is whether it has been drawn in a manner that perjury could be charged thereon and the affiant be held liable for damage caused. The oath required must refer to the truth of the facts within the personal knowledge of the applicant for search warrant, and/or his witnesses, not of the facts merely reported by a person whom one considers reliable. Tested by the above standard, the allegations of the witness do not come up to the level of facts of his personal knowledge so much so that he cannot be held liable for perjury for such allegations in causing the issuance of the questioned search warrant.

3. The rule is, that a description of a place to be searched is sufficient if the officer with the warrant can, with reasonable effort, ascertain and identify the place intended. In the case at bar, the application for search warrant and the search warrant itself described the place to be searched as the premises of PUP, and had specified the offices of the said university. The designation of the place to be searched sufficiently complied with the constitutional injunction that a search warrant must be particularly describe the place to be searched, even if there were several rooms at the ground floor and second floor of PUP.

4. Applicant’s failure to state under oath the urgent need for the issuance of the search warrant, his application having been filed on a Saturday, rendered the questioned warrant invalid for being violative of SC Circular 19 which provides that:

Case Digests: Searches and Seizures • Mark Justin Mooc

7 Burgos v. Chief of Staff

applications filed after office hours, during Saturdays, Sundays and holidays shall likewise be taken cognizance of and acted upon by any judge of the court having jurisdiction of the place to be searched, but in such cases, the applicant shall certify and state the facts under oath to the satisfaction of the judge that the issuance is urgent.

Guanzon v. De VillaThe military and police officers conducted “Areal Target Zonings” or “saturation drives” in Metro Manila, specifically on places where the subversives, as pinpointed by said authorities, were hiding. During these saturation drives, police and military units cordon an area of more than one residence and sometimes the whole barangay or areas of barangays, without any search warrant or warrant of arrest. Petitioners claimed that said saturation drives followed a common pattern of human rights abuses, as such, sought for its stoppage.

Facts:

1. The petitioners, who are of legal age, bona fide residents of Metro Manila, and taxpayers and leaders in their respective communities, sought to prohibit the military and police officers from conducting “Areal Target Zonings” or “saturation drives” in Metro Manila.

2. Petitioners claim that on various dates from March 5, 1987 till November 3 of the same year, various saturation drives were conducted by the respondents. Added by the petitioners, that these “saturation drives” are in critical areas pinpointed by the military and police as places where the subversives are hiding. The arrests ranged from 7 persons (July 20, Bankusay, Tondo) to 1,500 (November 3, Lower Maricaban, Pasay City) and that same followed a common pattern of human rights abuses like police and military units, without any search warrant or warrant of arrest, cordon an area of more than one residence and sometimes whole barangay or areas of barangay in Metro Manila, from the dead of the night or early morning hours and residents are herded as cows with men ordered to strip down to their briefs and examined for tattoo marks and other imagined marks.

Ruling:

1. There appears to have been no impediment to securing search warrants or warrants of arrest before any houses were searched or individuals roused from sleep were arrested. There is no strong showing that the objectives sought to be attained by the “areal zoning” could not be achieved as the rights of the squatter and low income families are fully protected. Where a violation of human rights specifically guaranteed by the Constitution is involved, it is the duty of the court to stop the transgression and state where even the awesome power of the state may not encroach upon the rights of the individual.

2. Where there is large scale mutiny or actual rebellion, the police or military may go in force to the combat areas, enter affected residences or buildings, round up

suspected rebels and otherwise quell the mutiny or rebellion without having to secure search warrants and without violating the Bill of Rights.

3. A show of force is sometimes necessary as long as the rights of the people are protected and not violated. A blanket prohibition such as that sought by the petitioners would limit all police power to one on one confrontation where search warrants and warrants of arrest against specific individuals are easily procured.

Pita v. Court of AppealsIn an Anti-Smut Campaign, members of the Metropolitan Police Force of Manila seized and confiscated along the sidewalks of Manila publications, magazines and other reading materials believed to be obscene, pornographic and indecent. One of said publications is “Pinoy Playboy” whose co-editor and publisher is Pita. Said materials were burned in public along U-Belt.

Facts:

1. Pursuing an Anti-Smut Campaign initiated by the Mayor of Manila (Ramon Bagatsing) on December 1 and 3, 1983, members of the Metropolitan Police Force of Manila seized and confiscated from dealers, distributors, newsstand owners and peddlers along Manila sidewalks magazines, publications and other reading materials believed to be obscene, pornographic and indecent. The said materials included “Pinoy Playboy” whose co-editor and publisher is the petitioner (Leo Pita). The said materials were burned in public along the University Belt along CM Recto Avenue, in the presence of Mayor Bagatsing and several officers and members of various student organizations.

2. Petitioner, on December 7, 1983, prayed for issuance of the writ of preliminary injunction against Mayor Bagatsing and the superintendent of the Western Police District of Manila (Narciso Cabrera), restraining them and their agents from confiscating Pinoy Playboy magazines or from preventing the sale of the said magazine for it, according to Pita, is a decent, artistic, and educational magazine.

3. Five days laters, petitioner filed an urgent motion for issuance of a TRO against indiscriminate seizure, confiscation and burning of the said magazine pending hearing on the petition for preliminary injunction.

4. In opposing petitioner’s application for a writ of preliminary injunction, Mayor Bagatsing pointed that during the anti-smut campaign, the materials confiscated belonged to the magazine stand owners and peddlers, who voluntarily surrendered their reading materials and that petitioner’s establishment was not raided.

5. The trial court denied the motion for a writ of preliminary injunction and dismissed the case for lack of merit. On appeal to the CA, RTC’s decision was affirmed.

Ruling:

1. It is basic that searches and seizures may be done only through a judicial warrant, otherwise, they become unreasonable and subject to challenge. Pertinent provisions state that the search must have been incident to a lawful search, and

Case Digests: Searches and Seizures • Mark Justin Mooc

8 Burgos v. Chief of Staff

the arrest must be on account of a crime committed. In the case at bar, no party has been charged, nor are such charges being readied against any party.

2. The Court rejected the argument that “there is no constitutional nor legal provision which would free the accused of all criminal responsibility because there had been no warrant” and that “violation of penal law must be punished.” For starters, there is no accused here to speak of, who out to be punished. Second, to say that the respondent Mayor could have validly ordered the raid (as a result of an anti-smut campaign), without a lawful search warrant because, in his opinion, “violation of penal laws” has been committed, is to make the respondent Mayor judge, jury and executioner rolled into one.

Valmonte v. De VillaThe NCR-District Command established checkpoints in various points of Valenzuela, Metro Manila. According to petitioner, who had been subjected to checkpoint once, the checkpoints caused worries among the residents of Valenzuela, especially the possibility of getting harassed.

Facts:

1. Activated through LOI 02/87 of the Philippine General Headquarters, AFP, the NCR District Command sought to conduct security operations within its area of responsibility and peripheral areas for the purpose of establishing an effective territorial defense, maintaining peace and order, and providing an atmosphere conducive to the social, economic and political development of the NCR. As part of its duty to maintain peace and order, the NCRDC installed checkpoints in various parts of Valenzuela, Metro Manila.

2. Petitioner (Ricardo Valmonte), together with the Union of Lawyers and Advocates for People’s Rights, contended that said checkpoints caused worries among the residents of Valenzuela, including the possibility of getting harassed. Aside from the possibility of getting harassed, residents worry of their safety due to the arbitrary, capricious and whimsical disposition of the military manning the checkpoints, considering that their cars and vehicles are being subjected to regular searches and check-ups, especially at night or at dawn, without the benefit of a search warrant and/or court order.

3. On July 9, 1988, a supply officer of the Municipality of Valenzuela, Bulacan (Benjamin Parpon) was gunned down (not killed) allegedly by members of the NCRDC manning the checkpoint for ignoring and/or refusing to submit himself to the checkpoint and for continuing to speed off in spite of warning shots fired in the air.

4. Petitioners (Valmonte and ULAP) contended that the said checkpoints give the respondents (De Villa) a blanket authority to make searches and/or seizures without search warrant or court order in violation of the Constitution. Valmonte has claimed that he had gone thru said checkpoints where he was stopped and his car subjected to search/check-up without a court order or search warrant.

Ruling:

1. No proof has been presented before the Court to show that, in the course of their routine checks, the military indeed committed specific violations of petitioners’ right against unlawful searches and seizures, or other rights. Petitioner’s general allegation that he had been stopped and searched without a search warrant by the military manning the checkpoints, without stating the details of the incidents which amount to a violation of his right against unlawful search and seizure, is not sufficient to enable the Court to determine whether there was a violation of Valmonte’s right against unlawful search and seizure.

2. The constitutional right against unreasonable searches and seizures is a personal right, and could be invoked only by those whose rights have been infringed or threatened to be infringed. What constitutes a reasonable or unreasonable search and seizure in any particular case is purely a judicial question, determinable from a consideration of the circumstances involved.

3. Not all searches and seizures are prohibited. Those which are reasonable are not forbidden. A reasonable search is not to be determined by any fixed formula but is to be resolved according to the facts of each case. When the officer merely draws aside the curtain of a vacant vehicle which is parked on the public fair grounds or simply looks into a vehicle or flashes a light therein, these do not constitute unreasonable search.

4. Between the inherent right of the State to protect its existence and promote public welfare and an individual’s right against a warrantless search, which is reasonably conducted, the former shall prevail.

People v. BurgosBurgos was alleged to be a member of the NPA. In his possession, one homemade revolver was found. He claimed that there was no valid warrant to effect search.

Facts:

1. Ruben Burgos was convicted for the crime of Illegal Possession of Firearms in Furtherance of Subversion. In his possession was found one homemade revolver, calibre .38, make Smith and Wesson. Said firearm was issued to and used by Burgos at Tiguman Digos, Davao de Sur by Alias Commander Pol of the NPA in the performance of his subversive tasks such as the recruitment of new members to the NPA and collection of contributions from the members.

2. Burgos, in his own account, claimed to have been torture and physical agony for he repeatedly refused to accept said firearm as his. He was undressed, with only blindfold, hot water poured in his body and over his private parts.

3. In his appeal to the SC, he claimed that there was no valid warrant to effect search in his house; thus, making him liable for the crime of illegal possession.

Ruling:

Case Digests: Searches and Seizures • Mark Justin Mooc

9 Burgos v. Chief of Staff

1. Based on the statement given by Cesar Masamlok (a former NPA), when the police authorities went to Burgos’ house, they did not have any warrant of arrest or search warrant with them.

2. Under Section 6(a) of Rule 113 which states that “[w]hen the person to be arrested has committed, is actually committing, or is about to commit an offense in his presence,” no search warrant or warrant of arrest is needed to make the arrest valid. Moreover, said offense must be committed in his presence or within his view. In the case at bar, there is no such personal knowledge in this case for whatever knowledge was possessed by the arresting officers came entirely from the information furnished by Cesar Masamlok. The location of the firearm was given by the Burgos’ wife. And, at the time of Burgos’ arrest, he was not in actual possession of any firearm or subversive document neither was he committing any act which could be described as subversive. In fact, he was plowing his field at the time of the arrest.

3. The right of a person to be secure against any unreasonable seizure of his body and any deprivation of his liberty is a most basic and fundamental one. The statute or rule which allows exceptions to the requirements of warrants of arrest is strictly construed. Any exception must clearly fall within the situations when securing a warrant would be absurd or is manifestly unnecessary as provided by the Rule. The court cannot liberally construe the rule on arrests without warrant or extend its application beyond the cases specifically provided by law. To do so would infringe upon personal liberty and set back a basic right so often violated and so deserving of full protection.

4. The questioned firearm and alleged subversive documents were obtained in violation of Burgos’ constitutional rights against unreasonable searches and seizures; this, making said articles inadmissible as evidence.

People v. MalmstedtOn Malmstedt’s way from Sagada to Angeles City, the police boarded the bus where he was riding. A bulge was spotted on Malmstedt’s waist and, when opened, 4 suspicious-looking objects wrapped in brown packing tape were found. When said objects were opened, the wrapped objects turned out to be hashish, a derivative of marijuana. Moreover, in each of his bags, teddy bears contained hashish.

Facts:

1. On May 11, 1989, the Commanding Officer (Capt. Alen Vasco) of the First Regional Command (NARCOM) ordered his men to set up a temporary checkpoint at Kilometer 14, Acop, Tublay, Mountain Province. Said checkpoint was for the purpose of checking all vehicles coming from the Cordillera Region and was prompted by persistent reports that vehicles coming from Sagada were transporting marijuana and other prohibited drugs. Moreover, the Commanding Officer received an information that a Caucasian (Mikael Malmstedt) coming from Sagada had in his possession prohibited drugs.

2. In the afternoon of same day, the bus where Malmstedt was riding was stopped. Malmstedt was on his way to Angeles City and would then proceed to Manila to catch his flight out of the country two days later. In the bus, 2 NARCOM officers (Sgt. Fider and CIC Galutan) boarded the bus and announced that they were members of the NARCOM and that they would conduct an inspection. Said officers started their inspection from the front going towards the rear of the bus where the accused was seated.

3. Galutan noticed a bulge on Malmstedt’s waist. He suspected that said bulge was a gun, thus he asked for the latter’s passport and other identification papers to which he (Malmstedt) failed to comply. For failure to comply with presenting passport and identification papers, Galutan required Malmstedt to bring out whatever it was that was bulging on his waist. It turned out that the bulging object was a pouch bag and when Malmstedt opened the said bag as ordered, the officer noticed 4 suspicious-looking objects wrapped in brown packing tape. When opened, the wrapped objects turned out to contain hashish, a derivative of marijuana.

4. Malmstedt was invited for questioning outside. But before leaving the bus, he stopped to get 2 travelling bags. The officers, upon Malmstedt’s alighting from the bus, got the bags and opened them. A teddy bear, having bulges, was found in each bag. After the bags were opened, it was then that Malmstedt presented his passport.

5. Malmstedt was brought to the headquarters of NARCOM at Camp Dangwas, La Trinidad, Benguet for further investigation. At the investigation room, the officers opened the teddy bears and found to contain hashish.

6. An information was filed against Malmstedt for violation of the Dangerous Drugs Act of 1972. Malmstedt raised the issue of illegal search of his personal effects.

Ruling:

1. There are exceptions where a search may be made pursuant to a lawful arrest which need not to obtain a search warrant. These circumstances include: (a) when the person to be arrested has committed, is actually committing or is attempting to commit an offense, in the presence of a peace officer or a private person; (b) when the offense was committed and the peace officer/private person has personal knowledge of facts indicating that the person to be arrested has committed it; and (c) when the person to be arrested is a prisoner who has escaped from a penal institution/place where he is serving final judgment or temporarily confined while his case is pending, or has escaped while being transferred from one confinement to another. In the case at bar, accused was searched and arrested while transporting prohibited drugs. A crime was actually being committed by the accused and he was caught in flagrante delicto. Thus, the search made upon his personal effects falls squarely under the first circumstance provided by the law which allow a warrantless search incident to a lawful arrest.

2. The acts of the NARCOM officers in requiring the accused to open his pouch bag and in opening one of the wrapped objects inside said bag as well as the two travel bags containing 2 teddy bears with hashish stuffed inside them, were prompted by

Case Digests: Searches and Seizures • Mark Justin Mooc

10 Burgos v. Chief of Staff

Malmstedt’s own attempt to hide his identity by refusing to present his passport, and by the information received by the NARCOM that a Caucasian coming from Sagada had prohibited drugs in his possession. To deprive the NARCOM agents of the ability and facility to act accordingly, including, to search even without warrant, in the light of such circumstance, would be to sanction impotence and ineffectiveness in law enforcement, to the detriment of society.

People v. Lo Ho Wing (alias Peter Lo), Lim Cheng Huat (alias Antonio Lim) and Reynaldo TiaReynaldo Tia, “a deep penetration agent” of the SOG, reported of his undercover activities on the suspected criminal syndicate led by Lo and Lim. Moreover, Tia informed his superior regarding their return to the country. Upon arrival in the Philippines, Lo and Tia rode in one taxi cab while Lim rode in another. They were pursued by the members of the NARCOM and were stopped. With permission of Lo and Tia, a tin can of tea was taken out of the red travel bag and, upon examination by the PC-INP Crime Laboratory, contained metamphetamine. Petitioner contend that a warrant was needed.

Facts:

1. The Special Operations Group received a tip from one of its informers about an organized group engaged in the importation of illegal drugs, smuggling of contraband goods and gunrunning. As part of the operations, the recruitment of confidential men and “deep penetration agents” was carried out to infiltrate the crime syndicate. One of those recruited was Reynaldo Tia.

2. Tia was introduced to Lim Cheng Huat (Antonio Lim) where the latter expressed a desire to hire a male travel companion for his business trips abroad. Tia offered his services and was hire. Together with Lim, Tia, in one of the meetings in China, was introduced to Lo Ho Wing (Peter Lo) whom tia found out to be the person he was to accompany to China in lieu of Lim.

3. As “deep penetration agent,” Tia regularly submitted reports of his undercover activities on the suspected criminal syndicate to Capt. Luisito Palmera, head of Oplan Sharon 887 – the group created in order to bus the suspected syndicate. Tia informed Palmera of their return to the Philippines after they (Lo and Tia) left for Hong Kong.

4. Upon arrival in the Philippines, they were met by Lim. After Lim and Lo finished their conversation, Lo hailed a taxicab. Lo and Tia boarded the taxicab while Lim followed in another taxi cab. Meanwhile, the operatives of the NARCOM (Narcotics Command), having been notified by Palmera, stationed themselves in strategic places around the arrival area. Upon seeing Lo and Tia leave the airport, the operatives followed them. Along Imelda Avenue, the car of the operatives overtook the taxicab ridden by Lo and Tia and cut into its path which forced the taxi driver to stop. The other tax cab carrying Lim, however, sped away but was later caught on Retiro Street, Quezon City.

5. Going back to Lo and Tia, the operatives approached the taxicab and asked the driver to open the baggage compartment. Three pieces of luggage were retrieved from the back compartment of the vehicle. The operatives requested from Lo and

Tia permission to search their luggage. A tin can of tea was taken out of the red travel bag owned by Lo. A certain Sgt. Cayabyab, one of the operatives, pried the lid open, pulled out a paper tea bag from the can and pressed it in the middle to feel its contents. Some crystalline white powder resembling crushed aluminium came out of the bag. The sergeant then opened the tea bag and examined its content more closely. He had the three travel bags opened for inspection. From the red travel bag, 6 tin cans were found, including the one previously opened and nothing else was recovered from the other bags.

6. The tea bag contained metamphetamine after examination by the PC-INP Crime Laboratory. One of metamphetamine’s derivatives is metamphetamine hydrochloride (shabu/poor man’s cocaine).

7. The three were charged with violation of Dangerous Drugs Act of 1972.8. Lo contends that the search and seizure was illegal. He contends that the officers

concerned could very well have procured a search warrant since they had been informed of the date and time of arrival of the accused at the NAIA well ahead of time. Moreover, as claimed by Lo, the fact that the search and seizure in question were made on a moving vehicle does not automatically make the warrantless search fall within the coverage of exceptions of the necessity of a valid warrant to effect search.

Ruling:

1. The search and seizure supported by a valid warrant is not an absolute rule. As set forth in Manipon, Jr. v. Sandiganbayan, there are at least 3 well-recognized exceptions, namely: (a) a search incidental to an arrest, (b) a search of a moving vehicle, and (c) seizure of evidence in plain view. In the case at bar, there is a clear showing that the search in question, having been made in a moving vehicle, does not need a valid warrant to effect search.

2. A warrantless search of a moving vehicle is justified on the ground that it is not practicable to secure a warrant because the vehicle can be quickly moved out of the locality or jurisdiction in which the warrant must be sought.

Yee Sue Kuy v. AlmedaA search warrant was issued, upon application by Almeda and presentation of Estrada as witness, to effect search and seizure of store and premises of Sam & Sing Co., which is owned by petitioner. Said search and seizure was in connection to petitioner’s activities of lending money at usurious rates.

Facts:

1. Respondent (Mariano Almeda), chief agent of the Anti-Usury Board, applied for a search warrant to command any peace officer to search during day time the store and premises occupied by Sam Sing & Co., situated at Sagay, Occidental Negros as well as the person of the said company, and to seize the documents, notebooks, lists, receipts and promissory notes. Said search warrant was issued by the justice

Case Digests: Searches and Seizures • Mark Justin Mooc

11 Burgos v. Chief of Staff

of peace of Sagay, Occidental Negros on the same day, May 5, 1938, after taking the testimony of Jose Estrada, a special agent of the Anti-Usury Board.

2. The said search warrant was to effect search and seizure of articles in connection with Sam Sing & Co.’s activities of lending money at usurious rates of interest, in violation of law.

3. The search warrant was enforced on the same day, at 10:30 a.m. by Almeda, Estrada, two internal revenue agents and two members of the Philippine Army. Immediately after the search and seizure, Almeda filed a return with the justice of peace of Sagay with a request that the office of the Anti-Usury Board be allowed to retain possession of the articles seized for examination, pursuant to Section 4, Act 4109.

4. Petitioner contended that the search warrant is illegal because the warrant was issued 3 days ahead of the application and Estrada’s affidavit is insufficient, and that seizure of the articles by means of a search warrant for the purpose of using them as evidence in the criminal case against the petitioners, is unconstitutional because the warrant becomes unreasonable and amounts to a violation of the constitutional prohibition against compelling the accused to testify against himself.

Ruling:

1. On the first contention, that is, issuance of search warrant 3 days prior to application, is not supported.

2. The criticism of petitioners that the search warrant in question was not issued in accordance with the formalities prescribed by Section 1, Paragraph 3 of Article III of the Constitution and of section 97, General Order 58 is unfounded. As a matter of fact, the strict observance of such formalities was followed. The applicant Almeda, in his application, swore that “he made his own personal investigation and ascertained that Sam Sing & Co. is lending money without license, charging usurious rate of interest and is keeping, utilizing and concealing in the store and premises, occupied by it documents, notebooks, lists, receipts, promissory notes and book of accounts and records. Moreover, witness Estrada, in his testimony before the judge, swore that he knew Sam Sing & Co. and its activities because he personally investigated the victims who secured loans from Sam Sing & Co.

3. The description of the articles seized, as given in the search warrant, is likewise sufficient. Where, by the nature of the goods seized, their description must be rather general, it is not required that a technical description be given, as this would mean that no warrant could issue.

4. Neither can there be objection to the fact that the objects seized from petitioners were retained by the agents of the Anti-Usury Board, instead of being turned over to the justice of the peace of Sagay, for the reason that the custody of said agents is the custody of the issuing officer/court, the retention having been approved by the latter.

Pasion vda. De Garcia v. LocsinDe Garcia’s person, house or store were subjected to a search by virtue of a search warrant. Said search was performed, together with the PC, by the agent of the Anti-Usury Board. Due to the confinement of petitioner due to an illness, the agent showed the search warrant to petitioner’s bookkeeper. Seized were two packages of records and a locked-filing cabinet containing several papers and documents.

Facts:

1. An agent of the Anti-Usury Board (Mariano Almeda) obtained from the justice of peace of Tarlac a search warrant commanding any officer of the law to search the person, house or store of petitioner at Victoria, Tarlac for “certain books, lists, chits, receipts, documents and other papers relating to her activities as usurer.”

2. On the same date (November 10, 1934), Almeda, together with the captain of the Philippine Constabulary, went to petitioner’s office in Victoria, Tarlac. After showing the search warrant to petitioner’s bookkeeper (Alfredo Salas), and without the presence of petitioner who was ill and confined at the time, Almeda proceeded with the warrant’s execution. Two packages of records and a locked filing cabinet containing several papers and documents were seized. Said papers and documents were kept for a considerable length of time by the Anti-Usury Board and were turned over by it (the Board) to the fiscal who filed 6 separate criminal cases against petitioner for violation of the Anti-Usury Law.

3. After the seizure, petitioner demanded the return of the documents seized. Moreover, the legality of the search warrant was challenged by the petitioner twice (January 7 and June 4, 1937).

Ruling:

1. Freedom from unreasonable searches and seizures is declared a popular right and for a search warrant to be valid, (a) it must be issued upon probable cause; (b) the probable cause must be determined by the judge himself and not by the applicant or any other person; (c) in the determination of probable cause, the judge must examine, under oath or affirmation, the complainant and such witnesses as the applicant may produce; and (d) the warrant issued must particularly describe the place to be searched and persons or things to be seized. In the case at bar, the existence of probable cause was determined not be the judge himself but by the applicant.

2. The constitutional immunity against unreasonable searches and seizures is a personal right which may be waived. The waiver may be either express or implied. It is well-settled that to constitute a waiver of constitutional right, it must appear that: (a) right exists, (b) persons involved had knowledge, either actual or constructive, of the existence of such right, and (c) said person had an actual intention to relinquish said right. The constitutional immunity from unreasonable searches and seizures, being a personal one, cannot be waived by anyone except the person whose rights are invaded or one who is expressly authorized to do so in

Case Digests: Searches and Seizures • Mark Justin Mooc

12 Burgos v. Chief of Staff

his/her behalf. In the case at bar, she could not have objected because she was sick and was not present when the warrant was served upon. Moreover, upon knowing of the seizure of some of her documents and papers, she had sent her lawyers to the office of the Anti-Usury Board to demand the return of the documents seized. The failure on the part of the petitioner and her bookkeeper to resist or object to the execution of the warrant does not constitute an implied waiver of constitutional right, rather it is merely a demonstration of regard for the supremacy of the law.

Burgos v. Chief of StaffThe newspaper offices of “Metropolitan Mail” and “We Forum” were searched, and office and printing macines, equipment, paraphernalia, motor vehicles and other articles used in printing, publication and distribution of said newspapers, among others, were seized. The premises were padlocked and sealed, which resulted to the newspapers’ discontinuance.

Facts:

1. Jose Burgos, Jr. is publisher-editor of the “We Forum” newspaper. Together with the “Metropolitan Mail” office, “We Forum” newspaper office was seized searched, and office and printing machines, equipment, paraphernalia, motor vehicles and other articles used in the printing, publication and distribution of said newspapers as well as numerous papers, documents, books and other written literature. Said articles were to be alleged to be in the possession and control of Burgos.

2. The search warrant was issued by Judge Ernani Cruz-Pano, issued last December 7, 1982. The application for the warrant was done by Col. Rolando Abadilla, Intelligence Officer of the PC Metrocom. Said application was accompanied by the Joint Affidavit by members of the Metrocom Intelligence and Security Group (Alejandro Gutierrez, Pedro Tango), both of whom were under Col. Abadilla and conducted a surveillance of the premises prior to the filing of the application for the warrant.

3. The search was televised in Channel 7 and widely publicized in all metropolitan dailies thus generating public interest. As a consequence of the search and seizure, said premises were padlocked and sealed, with the further result that the printing and publication of said newspapers were discontinued.

Ruling:

1. When the search warrant applied for is directed against a newspaper publisher or editor in connection with the publication of subversive materials, as in the case at bar, the application and/or its supporting affidavits must contain a specification, stating with particularity the alleged subversive material he has published or is intending to publish. Mere generalization would not suffice. Thus, the broad statement in Abadilla’s application that petitioner “is in possession or has in his control printing equipment and other paraphernalia, news publication, committing the offense of subversion punishable under PD 885 as amended” is a mere

conclusion of law and does not satisfy the requirements of probable cause. Bereft of such particulars as would justify a finding of the existence of probable cause, said allegation cannot serve as basis for the issuance of a search warrant.”

2. Section 2, Rule 126 of the Rules of Court enumerates the personal properties that may be seized under a search warrant, namely: (a) property subject of the offense, (b) property stolen or embezzled and other proceeds/fruits of the offense, and (c) property used or intended to be used as the means of committing an offense. Said rule does not require that the property to be seized should be owned by the person against whom the search warrant is directed. It may or may not be owned by him for under subsection (b), one of the properties that may be seized is stolen property. Stolen property must be owned by one other than the person in whose possession it may be at the time of the search and seizure. Ownership, therefore, is of no consequence and it is sufficient that the person against whom the warrant is directed has control or possession of the property sought to be seized, as petitioner was alleged to have in relation to the articles and property seized under the warrants.

Corro v. LisingCorro is the publisher and editor of the Philippine Times, whose offices were subjected to search and seizure of items and articles that were used and being used as instruments and means of committing the crime of inciting to sedition.

Facts:

1. Petitioner (Rommel Corro) is publisher and editor of the Philippine Times.2. Upon application filed by Lt. Col. Berlin Castillo of the PC-Criminal Investigation

Service, respondent (RTC Judge Esteban Lising) issued a search warrant on September 29, 1983, authorizing the search and seizure of: (a) printed copies of Philippine Times, (b) manuscripts/drafts of articles for publication in the Philippine Times, (c) newspaper dummies of the Philippine Times, (d) subversive documents, articles, printed matters, handbills, leaflets, banners, and (e) typewriters, duplicating machines, mimeographing and tape recording machines, video machines and tapes. Said items/articles were used and being used as instrument and means of committing the crime of inciting to sedition (Article 142).

3. On November 6, 1984, petitioner filed an urgent motion to recall warrant and to return documents/personal properties alleging, among others that said seized properties were not in any way connected with the offense of inciting to sedition and that the documents/papers seized has been rendered moot and academic due to the findings of the Agrava Board – having exclusive jurisdiction to determine the facts and circumstances behind the killing of Ninoy Aquino – that a military conspiracy was responsible for Ninoy Aquino’s slaying. Said motion was denied by respondent.

Ruling:

Case Digests: Searches and Seizures • Mark Justin Mooc

13 Burgos v. Chief of Staff

1. Probable cause, as defined in Burton v. St. Paul, M&M. Ry. Co., is constituted by “such reasons, supported by facts and circumstances, as will warrant a cautious man in the belief that his actions, and the means taken in prosecuting it, are legally just and proper.” Thus, an application for search warrant must state with particularity the alleged subversive materials published or intended to be published by the petitioner.

2. A search warrant should particularly describe the place to be searched and the things to be seized. The evident purpose and intent of this requirement is to limit the things to be seized to those, and only those, particularly described in the search warrant – to leave the officers of the law with no discretion regarding what articles should they should seize, to the end that unreasonable searches and seizures may not be committed.

3. The statement of Col. Castillo in his affidavit state that they have “found that the said publication in fact foments distrust and hatred against the government of the Philippines and its duly constituted authorities,” together with Lt. Ignacio’s statement that said periodical “contains articles tending to incite distrust and hatred for the Philippine Government,” is a mere conclusion of law and would not satisfy the requirements of probable cause.

Olaes v. PeopleOlaes was indicted for violation of Dangerous Drugs Act of 1972. He was believed to have in his possession marijuana dried stalks/leaves/seeds/cigarettes and other regulated/prohibited and exempt narcotics preparations.

Facts:

1. Adolfo Olaes was believed to have in his possession marijuana dried stalks/leaves/seeds/cigarettes and other regulated/prohibited and exempt narcotics preparations; thus, indicting petitioners of violation of RA 6425 (Dangerous Drugs Acts of 1972) despite failure to pinpoint specific section of same.

2. Petitioners challenged the admission of evidence seized by virtue of an allegedly invalid warrant issued on March. More so, petitioners claimed that the search warrant issued by the judge is unconstitutional because it did not indicate the specific offense the petitioners have supposedly committed; thus, making no valid finding of probable cause as a justification for the issuance of the said warrant in conformity with the Bill of Rights.

Ruling:

1. Although the specific section of the Dangerous Drugs Act is not pinpointed, there is no question at all of the specific offense alleged to have been committed as a basis for the finding of probable cause. The search warrant also satisfies the requirement in the Bill of Rights of the particularity of the description to be made of the “place to be searched and the persons or things to be seized.” Thus, the articles seized under the challenged search warrant were admitted as evidence.

Presidential Anti-Dollar Salting Task Force v. CAKaramfil Import-Export Co., Inc, together with other enterprises, were subjected to search by virtue of 6 search warrants, having been applied for by a particular Atty. Gatmaytan. PADS Task Force issued said search warrants.

Facts:

1. The PADS Task Force through State Prosecutor Jose Rosales issued 6 search warrants against Karamfil Import-Export Co, Inc., P&B Enterprise Co., Inc., Philippine Veterans Corporation, Philippine Veterans Development Corporation, Philippine Construction Development Corporation, Philippine Lauan Industries Corporation, Inter-Trade Development, Amelili U. Malaquiok Enterprises and Jaime P. Lucman Enterprises. Said search warrants were issued upon application by Atty. Napoleon Gatmaytan of the Bureau of Customs and a deputized member of the PADS Task Force, together with the affidavit of Josefin M. Castro, an operative and investigator of the PADS Task Force.

2. Respondents questioned whether the PADS Task Force is “such other responsible officer” allowed/countenanced by the 1973 Constitution to issue warrants of search and seizures. The RTC, therefore, declared the said search warrants as null and void, and eventually denied reconsideration. In disposing of the petition, the said court found the material issues to include: (a) competency of RTC to act on petition filed by the petitioners, (b) validity of the search warrants issued by the respondent State Prosecutor, and (c) whether the petition has become moot and academic because all the search warrants sought to be quashed had already been implemented and executed.

3. On appeal, PADS was upheld. The CA declared that the PADS Task Force is a quasi-judicial body, making it co-equal with the RTC. However, on motion for reconsideration by Karamfil, the CA reversed itself.

Ruling:

1. The PADS, as stated in the task force’s organic act PD 1936 as amended by PD 2002, was not meant to exercise quasi-judicial functions to try and decide claims and execute its judgment. It is the President’s arm called upon to combat the vie of “dollar salting” or the blackmarketing and salting of foreign exchange. It is rather tasked by the PD to handle the prosecution of such activities but nothing more. Thus, not being a quasi-judicial body, it cannot be considered co-equal or coordinate with RTC.

2. Under the 1887 Constitution, the powers of arrest and search are exclusive upon judges. The incident, which happened during the effectivity of the 1973 Constitution, had become moot and academic.

3. When the 1973 Constitution spoke of “responsible officer” to whom the authority to issue arrest and search warrants may be delegated by legislation, it did not furnish the legislator with the license to give that authority to whomsoever it pleased. It is to be noted that the Charter qualified that the officer himself must be

Case Digests: Searches and Seizures • Mark Justin Mooc

14 Burgos v. Chief of Staff

“responsible.” The Court takes “responsibility,” as used by the Constitution, to mean not only skill and competence but more significantly, neutrality and independence comparable to the impartiality presumed of a judicial officer. Thus, a prosecutor falls short to be considered having possessed the latter qualities. The implied exclusion of prosecutors under the 1973 Constitution was founded on the requirements of due process, specifically the assurance to the respondent of an unbiased inquiry of the charges against him prior to the arrest of his person or seizure of his property.

4. The Court agreed that the PADS Task Force is meant to exercise prosecutorial powers, and on that ground, it cannot be said to be a neutral and detached “judge” to determine the existence of probable cause for purposes of arrest or search.

Salazar v. AchacosoSalazar’s properties in her residence and dance studio were seized by virtue of a search warrant issued by the POEA.

Facts:

1. Petitioner (Hortencia “Horty” Salazar) was charged by a Rosalie Tesoro wth the Philippine Overseas Employment Administration. According to Tesoro, after she surrendered her PECC Card to petitioner, she promised her of “booking” in Japan. However, after 9 months, Tesoro was still in the Philippines and was never able to travel to Japan, and that her PECC card was not released by Salazar.

2. Public respondent Atty. Ferdinand Marquez sent a telegram to petitioner. Respondent requested the petitioner before him being a part of POEA Anti-Illegal Recruitment Unit. On the same day, having ascertained that the petitioner had no license to operate a recruitment agency, administrator Tomas Achacoso issued a closure and seizure order, numbered 1205.

3. The Director of POEA Licensing and Regulation (Atty. Estelita Espiritu) issued an order designation Atty. Marquez, Atty. Abara and Atty. Vistro as members of the team tasked to implement the Closure and Seizure Order rendered by Achacoso. After proceeding to petitioner’s residence, the team, assisted by Mandaluyong policemen and mediamen, went to Hannalie Dance Studio, which petitioner operated.

4. Before entering Hannalie Dance Studio, the team served said order on a certain Mrs. Flora Salazar who voluntarily allowed them entry into the premises. When required to show credentials, Salazar was unable to produce any. The team confiscated assorted costumes when they chanced upon 12 talent performers practicing a dance number. The confiscation was duly receipted for by Mrs. Asuncion Maguelan and witnessed by Salazar.

5. Petitioner, through a letter to POEA, requested that the personal properties seized at her residence be returned.

Issue: Whether the POEA validly issue warrants of search and seizure (or arrest) under Article 38 of the Labor Code.

Ruling:

1. Under the present Constitution, it is only a judge who may issue warrants of search and arrest. It was declared that mayors may not exercise this power, neither by a mere prosecuting body. The exception is in cases of deportation of illegal and undesirable aliens, whom the President or the Commissioner of Immigration may order arrested, following a final order of deportation, for purpose of deportation.

2. Section 38(c), as amended by PD 1920 and 2018, bestowed to the Minister of Labor the power to recommend the arrest and detention of any person engaged in illegal recruitment. More so, PD 1920 gave the Minister of Labor arrest and closure power. That, the Minister of Labor and Employment has the power to cause the arrest and detention of such non-licensee or nonholder of authority if after proper investigation it is determined that his activities constitute a danger to national security and public order or will lead to further exploitation of job-seekers. Meanwhile, PD 2018 bestowed upon the Minister of Labor search and seizure powers. However, the decrees in question stood as dying vestiges of authoritarian rule in its twilight moments. Thuss, the Secretary of Labor, not being a judge, may no longer issue search or arrest warrants. Article 38(c) of the Labor Code is declared unconstitutional and of no force and effect.

Stonehill v. Diokno42 search warrants were issued by judges, upon application by officers of government. 29 out of the 42 search warrants were intended for the corporations and offices which the petitioners were affiliated with; the rest were intended for their residences. Petitioners were charged for violating the Central Bank Laws, Tariff and Customs Law, Internal Revenue and RPC. In the performance of the search warrants, several items were seized including books of accounts, financial records and documents showing all business transactions.

Facts:

1. A total of 42 search warrants were issued against petitioners and/or corporations of which they were officers by several judges upon the application of the officers of government (Diokno as Secretary of Justice, Jose Lukban as Acting Director of NBI, among others). Said search warrants directed any peace officer to search the persons of petitioners (Harry Stonehill, Robert Brooks, John Brooks, Karl Beck) and/or the premises of their offices, warehouses and/or residences, and to seize and take possession of personal property, which includes: books of accounts, financial records, vouchers, correspondence, receipts, ledgers, journals, portfolios, credit journals, typewriters, and other documents and/or papers showing all business transactions including disbursements receipts, balance sheets and profit and loss statements and Bobbins (cigarette wrappers). Said items/articles are “the subject of the offense, stolen or embezzled and proceeds/fruits of the offense” or “used or intended to be used as the means of committing the offense,” which is violation of Central Bank Laws, Tariff and Customs Laws, Internal Revenue, and the RPC.

Case Digests: Searches and Seizures • Mark Justin Mooc

15 Burgos v. Chief of Staff

2. Petitioners contend that the search warrants are null and void for: (a) they do not describe with particularity the documents, books and things to be seized. (b) cash money, not mentioned in the warrants, were actually seized, (c) the warrants were issued to fish evidence against the aforementioned petitioners in deportation cases filed against them, (d) the searches and seizures were made illegally, and (e) the documents, papers and cash money seized were not delivered to the courts that issued the warrants.

3. Respondents, in their answer, alleged that the contested search warrants are valid and have been issued in accordance with law, that the defects of said warrants, if any, were cured by petitioners’ consent, and that the effects seized are admissible in evidence against petitioners, regardless of the alleged illegality of the aforementioned searches and seizures.

Ruling:

1. In deciding this case, the Court split the documents, papers, and things seized into two major groups: (a) those found and seized in the offices of the aforementioned corporations and (b) those found and seized in the residences of petitioners.

2. With regard to the first group, i.e., those found and seized in the offices of the aforementioned corporations, petitioners have no cause of action to assail the legality of the contested warrants and of the seizures made pursuant thereof. It is for the reason that said corporations have their respective personalities, separate and distinct from the personality of petitioners, regardless of the amount of shares of stock or of the interest and whatever office they may hold. The legality of the seizure can be contested only by the party whose rights have been impaired and that the objection to an unlawful search and seizure is purely personal and cannot be availed of by third parties. Petitioners may not validly object to the use of articles seized from the offices as evidence against them since the right to object to the admission of said papers in evidence belongs exclusively to corporation to whom the seized effects belong and may not be invoked by the corporate officers in proceedings against them in their individual capacity.

3. As for the second group, i.e., those found and seized in petitioners’ residences, said items/articles cannot be used as evidence against them. None of the requirements laid down by the Constitutional provision (that no warrant shall issue but upon probable cause, to be determined by the judges in the manner set forth in said provision, and that the warrant shall particularly describe the things to be seized) has been complied with in the contested warrants. No specific offense has been alleged in said applications for the said applications stated that the persons concerned have violated Central Bank Laws, Tariff and Customs Laws, Internal Revenue and RPC. As a consequence, it was impossible for the judges who issued the warrants to have found existence of probable cause. More so, the applications did not allege any specific act performed by petitioners.

4. The constitutional provision on searches and seizures seek to outlaw general warrants. More so, no search warrant shall issue for more than one specific offense.

5. Respondents, citing Moncado v. People’s Court, maintained that, despite the unconstitutionality of the searches and seizures, the items/articles seized are admissible in evidence against petitioners. However, said doctrine is abandoned.

6. The non-exclusionary rule is contrary both to the letter and spirit of the constitutional injunction against unreasonable searches and seizures. (The non-exclusionary rule is that established in Moncado v. People’s Court. ) To be sure, if the applicant for a search warrant has competent evidence to establish probable cause of the commission of a given crime by a party against whom the warrant is intended, there is no reason why the applicant should not comply with the requirements of the fundamental law. Upon the other hand, if he has no such competent evidence, then it is not possible for the judge to find that there is probable cause, and, hence, no justification for the issuance of the warrant. The only possible explanation for its issuance is the necessity of fishing evidence of the commission of a crime. But, this fishing expedition is indicative of the absence of evidence to establish a probable cause.

7. The search warrants of petitioners’ residences (group 2) are null and void. As for the warrants in 29 places, offices and other premises (group 1), they are valid.

Papa v. MagoPapa, Chief of Police of Manila and a duly deputized member of the Bureau of Customs, together with Alagao and other elements of the counter-intelligence unit, seized 9 bales of goods from two trucks. Said items, according to an information, were misdeclared and undervalued. The cargo owner, respondent in this case, claimed that the MPD seized said goods without a search warrant.

Facts:

1. Petitioner Martin Alagao (head of the counter-intelligence unit of the MPD), having received a reliable information that a certain shipment of personal effects were allegedly misdeclared and undervalued and were to be released from the customs zone of the port of Manila, conducted surveillance of said zone. With him were petitioner Ricardo Papa, the Chief of Police of Manila and a duly deputized member of the BOC, and other elements of the counter-intelligence unit. The information which reached Alagao specified that said misdeclared and undervalued items were loaded on two trucks.

2. The trucks left the gate where Alagao’s group conducted surveillance. However, such trucks were later intercepted. The load of the two trucks consisted of 9 bales of goods.

3. The cargo was owned by Remedios Mago while the truck was owned by Valentin Lanopa. In their petition in the CFI of Manila, they claimed that the MPD seized the goods without search warrant issued by a competent court, and that Papa denied the request of Mago’s counsel that the bales be not opened and the goods not examined.

Case Digests: Searches and Seizures • Mark Justin Mooc

16 Burgos v. Chief of Staff

4. The respondent judge issued an order restraining petitioners from opening the nine bales in question. However, some bales were already opened by examiners of the BOC when the restraining order was received.

5. Respondent contended that, since the inventory of the goods seized did not show any article of prohibited importation, such articles should be released upon her posting of the bond to be determined by court. Petitioners contended however that most of the goods, as shown in the inventory, were not declared and were thus subject to forfeiture. Respondent judge issued an order releasing the good upon the filing of the bond in the amount of Php 40,000.00 to which the respondent complied with.

Issue: Is there a need to procure a warrant before search be made?

Ruling:

1. The Bureau of Customs acquires exclusive jurisdiction over imported goods, for the purposes of enforcement of the customs laws, from the moment the goods are actually in possession or control, even if no warrant of seizure or detention had previously been issued by the Collector of Customs in connection with seizure and forfeiture proceedings. In the case at bar, the moment the BOC actually seized the goods in question, the BOC acquired jurisdiction over the goods for the purposes of enforcement of the tariff and customs laws, to the exclusion of the regular courts.

2. Petitioner Alagao and his companion policemen had authority to effect the seizure without any search warrant issued by a competent court. The Tariff and Customs Code does not require said warrant in the instant case. The Code authorizes persons having police authority under Section 2203 to enter, pass through or search any land, inclosure, warehouse, store or building, not being a dwelling house; and also to inspect, search and examine any vessel or aircraft and any trunk, package or envelope or any person on board, or to stop and search and examine any vehicle, beast or person suspected of holding or conveying any dutiable or prohibited articles.

Nolasco v. Cruz-PañoAguilar-Roque is accused of rebellion. On August 6, 1984, she was arrested together with Nolasco. 30 minutes after her arrest, the vicinity where she was arrested was likewise searched. During said search, 431 items were seized and the person in-charge of the premises, Tolentino, was arrested. Petitioners assert that the search warrant partake of a general warrant; thus, said items cannot be admitted as evidence.

Facts:

1. One of the petitioners (Aguilar-Roque) was accused of the Rebellion of Military Commission No. 25. She was arrested on August 6, 1984, 11:30 AM by a Constabulary Security Group (CSG). Arrested with Roque was Nolasco. 30 minutes later, elements of the CSG searched the premises at 239-B Mayon St., Quezon City. During the said search, one of the petitioners Tolentino, who was the person in-

charge of the premises, was arrested. 428 documents and written materials were seized, together with a portable typewriter and 2 wooden boxes.

2. Three hours prior to the search, Lt. Col. Virgilio G. Saldajeno of the CSG applied for a search warrant from Judge Paño to be served on 239-B Mayon St., Quezon City. Said place was determined to be the leased residence of Aguilar-Roque after almost a month of “round the clock surveillance.” Said warrant was issued in proceedings entiled “PP v. Mila Aguilar-Roque, Accused, Search Warrant No. 80-84 for rebellion.” This is known to be the Search Warrant Case.

3. Nolasco, Aguilar-Roque and Tolentino wre charged for subversion/rebellion and/or conspiracy to commit rebellion/subversion.

4. Petitioners, on December 12, prayed in a Motion to Suppress filed with MTC Judge Santos that the items (total of 431) be returned to them. Such motion was denied by Judge Santos on the ground that the validity of the Search Warrant had to be litigated in the Search Warrant Case.

5. Petitioners assert that the search warrant is void because it is a general warrant since it did not sufficiently describe with particularity the things subject of the search and seizure and that probable cause had not been properly established for lack of searching questions.

Ruling:

1. The items enumerated in the search warrant were vaguely described and not particularized. There is absent a definite guideline to the searching team as to what items might be lawfully seized this giving the officers of the law discretion regarding what articles they should seize. Therefore, it is in the nature of a general warrant and thus infringes the constitutional mandate requiring particular description of the things to be seized.

2. Notwithstanding the irregular issuance of the search warrant and although, ordinarily, the articles seized under an invalid search should be returned, they cannot be ordered returned in the case at bar, for some searches may be made without warrant. As declared in Section 12, Rule 126 of the Rules of Court, a person charged with an offense may be searched for dangerous weapons or anything which may be used as proof of the commission of the crime. Said provision is confined to search, without a search warrant of a person who had been arrested. It is also a general rule that, as an incident of an arrest, the place or premises where the arrest was made can also be searched without a search warrant. In the latter case, “the extent and reasonableness of the search must be decided on its own facts and circumstances, and it has been stated that, in the application of general rules, there is some confusion in the decisions as to what constitutes the extent of the place or premises which may be searched. What must be considered is the balancing of the individual’s right to privacy and the public’s interest in the prevention of crime and the apprehension of criminals.”

3. Roque –charged with rebellion which is a crime against public order, a warrant for her arrest had not been served for a considerable period of time, arrested within the general vicinity of her dwelling, and search of her dwelling was made within a

Case Digests: Searches and Seizures • Mark Justin Mooc

17 Burgos v. Chief of Staff

half hour of her arrest – did not need a search warrant for the possible effective results in the interest of public order.

Posadas v. CAPetitioner was walking within the premises of Rizal Memorial Colleges when he was spotted by 2 members of the INP. He was spotted carrying a buri bag and, according to the INP members, was acting suspiciously. When he was approached by the officers who duly identified themselves as members of the INP, petitioner attempted to flee but was stopped. The buri bag, when checked, contained a calibre .38 gun, ammunitions for a .38 calibre and a .22 calibre gun, and a smoke grenade.

Facts:

1. Petitioner was caught during the surveillance of members of the Integrated National Police (Ursicio Ungab and Umbra Umpar) on October 16, 1986 at about 10 in the morning. He was caught in Magallanes St., Davao Citym within the premises of Rizal Memorial Colleges. Petitioner was carrying a buri bag and “was acting suspiciously,” as described by the two members of the INP.

2. When they approached petitioner, they identified themselves as members of the INP. Petitioner attempted to flee but was thwarted. The two officers checked the buri bag and found: 1 caliber .38 Smith & Wesson revolver with serial no. 770196, 2 rounds of live ammunition for a .38 caliber gun, a smoke grenade, and 2 live ammunitions for a .22 caliber gun.

3. Petitioner was brought to the headquarters and was asked to show the necessary license or authority to possess firearms and ammunitions found in his possession, but correspondingly failed to do so. He was convicted for illegal possession of firearms and ammunitions.

4. Petitioner contends however that, there being no lawful arrest or search or seizure, the items which were confiscated from his possession were inadmissible as evidence against him.

Ruling:

1. Section 5, Rule 113 of the 1985 Rules on Criminal Procedure provides that, among others, an arrest is lawful even without a warrant should a person has committed, is actually committing, or is attempting to commit an offense in the presence of a peace officer or a private person. In the case at bar, the officers did not know what the petitioner had committed or was actually committing; thus, it does not justify an arrest without a warrant. However, the search thereat in the case at bar is more reasonable than warrantless search and seizure conducted at military or police checkpoints. The search done by the officers was effected on the basis of a probable cause. The probable cause is that when the petitioner acted suspiciously and attempted to flee with the buri bag. There was a probable cause that he was concealing something illegal in the bag and it was the right and duty of the police officers to inspect the same.

2. It is too much to require police officers to search bags in the possession of the petitioner only after they shall have obtained a search warrant for the purpose. Such an exercise may prove to be useless, futile and much too late.

People v. de LaraSurveillance was conducted on December 15 and 17, 1986 and January 8, 1987 on the vicinity where de Lara was captured on January 9. One of the team to execute the buy-bust operation acted as the poseur-buyer. Replying to the question of de Lara, said poseur-buyer ordered 2 foils and handed the marked Php 20 bill. When de Lara, after handing the two foils to the poseur-buyer, sensed the presence of the police, he ran inside the house. He was pursued by the poseur-buyer and, when subdued by the operatives, de Lara admitted that he kept prohibited drugs in his house and even showed a plastic containing prohibited drugs.

Facts:

1. On January 9, 1987, after surveillance of the vicinity of Garrido and Zamora Sts. At Sta. Ana, Manila last December 15 and 17 of 1986, and January 8, 1987, a six-man team was formed in order to execute a buy-bust operation against de Lara and his group. A certain Pfc. Martin Orolfo, Jr. acted as the poseur-buyer.

2. Orolfo and the confidential informant proceeded to the house of de Lara where he was seen standing outside. The informant introduced Orolfo as an interested buyer of marijuana, to which de Lara asked how much he (Orolfo) would buy. Responding to the question, Orolfo answered two foils handing at the same time the marked Php 20 bill. de Lara placed the money in the right pocket in his pants,, went inside and minutes later came back with the two foils.

3. When de Lara handed the two foils, he sensed the presence of the police; thus, he tried to retrieve the two foils from Orolfo to which the latter prevented him from doing so. He ran inside the house, with Orolfo in pursuit. When he was subdued, de Lara admitted that he kept prohibited drugs in his house and even showed the arresting officers a blue plastic bag with white lining containing prohibited drugs. Orolfo made a receipt of the articles seized.

4. de Lara was convicted of violation of Sec. 4, Article II of RA 6425 (Dangerous Drugs Act of 1972). In his appeal, he questioned the legality of his arrest and seizure of prohibited drugs found in his house.

Ruling:

1. Section 5, Rule 113 of the 1985 Rules on Criminal Procedures enumerates situations when an arrest may be lawful even without a warrant. Two of said situations applicable to the case are: (a) that when the person to be arrested has committed, is actually committing or is attempting to commit an offense in the presence of a peace officer/private person, and (b) that when an offense has in fact just been committed and the peace officer/private person has personal knowledge of facts indicating that the person to be arrested has committed it. In the case at bar, de Lara was caught red-handed in delivering two tin foils of marijuana to

Case Digests: Searches and Seizures • Mark Justin Mooc

18 Burgos v. Chief of Staff

Orolfo. Having caught the appellant in flagrante as a result of the buy-bust operation, the policemen were not only authorized but were also under obligation to apprehend the drug pusher even without a warrant of arrest. Furthermore, surveillance on the illegal activities of de Lara was already conducted by the police as early as December 15 and 17, 1986.

2. The policemen’s entry into the house of appellant without a search warrant was in hot-pursuit of a person caught committing an offense in flagrante. The arrest that followed the hot-pursuit was valid.

3. Moreover, the seizure is valid. The seizure of the plastic bag containing prohibited drugs was the result of appellant’s arrest inside his house. A contemporaneous search may be conducted upon the person of the arrestee and the immediate vicinity where the arrest was made.

People v. de GraciaThere was a coup d’ etat staged from November 30 to December 9, 1989. A surveillance was conducted on the night of November 30 till the early morning of December 1 on Eurocar Sales Office located in EDSA. The surveillance team was attacked by five men coming from the Eurocar building. On December 5, the building was raided and de Gracia, together with the janitors of the building, was caught. Found in his possession were high-powered firearms, ammunitions and explosives.

Facts:

1. From November 30 to December 9, 1989, there was a coup d’ etat staged by elements of the Reform the Armed Forces Movement – Soldiers of the Filipino People (RAM-SFP). Various government establishments and military camps in Metro Manila were bombarded by the RAM-SFP with their “tora-tora” planes.

2. On the night of November 30, 1989 until the early morning of the next day, Major Efren Soria of the Intellience Division conducted a surveillance of the Eurocar Sales Offices at EDSA, together with his team. They were informed that said establishment were being occupied by elements of the RAM-SFP as a communication command post. One member of Soria’s team (S/Sgt. Henry Aquino) conducted a surveillance on foot when the crowd gathered near the Eurocar Office watching the on-going bombardment near Camp Aguinaldo and from said crowd, a group of five men walked towards the car of the surveillance team. When the vehicle sped away, the group of five men fired at the team which resulted in the wounding of a team member (Sgt. Sagario).

3. On December 5, 1989, a searching team led by F/Lt. Virgilio Babao, together with the elements of the 16th Infantry Battalion led by Col. Delos Santos, raided the Eurocar Sales Office and found ammunitions and explosives. A member of the team, Sgt. Obenia, who was the first one to enter the building, saw de Gracia holding a C-4 and suspiciously peeping through a door. de Gracia was arrested, together with the janitors of the building. They were made to sign an inventory, written in Tagalog, of the explosives and ammunition confiscated by the raiding team. No search warrant was secured by the raiding team because, according to

them, there was so much disorder considering that Camp Aguinaldo was being mopped up by the rebel forces and there was simultaneous firing within the vicinity of the Eurocar Office, aside from the fact that courts were consequently closed.

4. Rolando de Gracia was charged with two separate informations for illegal possession of ammunition and explosive in furtherance of rebellion and for attempted homicide. Found in their possession were 5 bundles of dynamites, 6 cartons of M16 ammunition at 20 per carton and 100 bottles of MOLOTOV bombs. de Gracia was convicted for the first crime (furtherance of rebellion) but was acquitted of the second (of attempted homicide).

Ruling:

1. It is admitted that the military operatives who raided the Eurocar Sales Office were not armed with a search warrant at that time. Said search was prompted by intelligence reports that said office was being used as headquarters by the RAM_-SFP. Prior to the raid, there was a surveillance conducted on the premises wherein the surveillance team was fired at by a group of men coming from the Eurocar Office. When the military operatives raided the place, the occupants refused to open the door despite requests for them to do so, thereby compelling the military to break into the office. The Eurocar Sales Office is neither a gun store nor an armory or arsenal; instead, it was primarily and solely engaged in the sale of automobiles. The presence of an unusual quantity of high-powered firearms and explosives could not be justifiably or colorably explained. In addition, there was general chaos and disorder at that time. The courts in the surrounding areas wre obviously closed with the building and houses deserted. Under said circumstances, the case at bar falls under one of the exceptions to the prohibition against a warrantless search. In the first place, the military operatives had reasonable ground to believe that a crime was being committed after taking into account the facts. More so, there is more than sufficient probable cause warrant their action. Under the situation then prevailing, the raiding team had no opportunity to apply for and secure a search warrant from the courts. The judge himself manifested that when the raid was conducted his court was closed. Under such urgency and exigency of the moment, a search warrant could lawfully be dispensed with.

2. As enunciated in Umil, et.al v. Ramos, “[t]he arrest of persons involved in the rebellion… is more an act of capturing them in the course of an armed conflict, to quell the rebellion than for the purpose of immediately prosecuting them in court for a statutory offense. The arrest, therefore, need not follow the usual procedure in the prosecution of offenses which requires the determination by a judge of the existence of probable cause before the issuance of a judicial warrant of arrest… Obviously, the absence of a judicial warrant is no legal impediment to arresting or capturing persons committing overt acts of violence against government forces or any other milder acts but really in pursuance of the rebellious movement. The arrest or capture is thus impelled by the exigencies of the situation that involves the very survival of society and its government and duly constituted authorities.”

Case Digests: Searches and Seizures • Mark Justin Mooc